Difference between revisions of "1984 AHSME Problems/Problem 5"

m (Solution)
m
(2 intermediate revisions by 2 users not shown)
Line 9: Line 9:
 
==See Also==
 
==See Also==
 
{{AHSME box|year=1984|num-b=4|num-a=6}}
 
{{AHSME box|year=1984|num-b=4|num-a=6}}
 +
{{MAA Notice}}

Revision as of 09:49, 8 November 2016

Problem 5

The largest integer $n$ for which $n^{200}<5^{300}$ is

$\mathrm{(A) \ }8 \qquad \mathrm{(B) \ }9 \qquad \mathrm{(C) \ } 10 \qquad \mathrm{(D) \ }11 \qquad \mathrm{(E) \ } 12$

Solution

Since both sides are positive, we can take the $100th$ root of both sides to find the largest integer $n$ such that $n^2<5^3$. Fortunately, this is simple to evaluate: $5^3=125$, and the largest square less than $125$ is $11^2=121$, so the largest $n$ is $11, \boxed{\text{D}}$.

See Also

1984 AHSME (ProblemsAnswer KeyResources)
Preceded by
Problem 4
Followed by
Problem 6
1 2 3 4 5 6 7 8 9 10 11 12 13 14 15 16 17 18 19 20 21 22 23 24 25 26 27 28 29 30
All AHSME Problems and Solutions

The problems on this page are copyrighted by the Mathematical Association of America's American Mathematics Competitions. AMC logo.png